Vector space is decomposed into direct sum of its subspace and its orthogonal complement












2












$begingroup$


Let $E$ be a finite-dimensional vector space over the field $k$ and let $g$ be a bilinear form which is symmetric, antisymmetric or hermitian. Let $V$ be a subspace of $E$. Let $V_1 = { x| x in E, g(x,a)=0 forall a in V}$. Prove that $E$ is a direct sum of $V$ and $V_1$. This result is used in the proof as well-known but unfortunately I can't find the proof of it. The proof I know is in the case of $g$ being inner product.










share|cite|improve this question











$endgroup$








  • 2




    $begingroup$
    Well, if you can't "find the proof" come up with your own! Or at least try...
    $endgroup$
    – DonAntonio
    Nov 7 '13 at 20:29






  • 1




    $begingroup$
    This is false as stated, you need some kind of non-deneracy hypothesis on $g$. For example, if $g(x,y) = 0$ for all $x$ and $y$, then $g$ is symmetric. Now, let $V = E$. Then $V_1 = E$, but $Enotcong Eoplus E$ (for finite dimensional vector spaces).
    $endgroup$
    – Jason DeVito
    Oct 14 '15 at 14:18


















2












$begingroup$


Let $E$ be a finite-dimensional vector space over the field $k$ and let $g$ be a bilinear form which is symmetric, antisymmetric or hermitian. Let $V$ be a subspace of $E$. Let $V_1 = { x| x in E, g(x,a)=0 forall a in V}$. Prove that $E$ is a direct sum of $V$ and $V_1$. This result is used in the proof as well-known but unfortunately I can't find the proof of it. The proof I know is in the case of $g$ being inner product.










share|cite|improve this question











$endgroup$








  • 2




    $begingroup$
    Well, if you can't "find the proof" come up with your own! Or at least try...
    $endgroup$
    – DonAntonio
    Nov 7 '13 at 20:29






  • 1




    $begingroup$
    This is false as stated, you need some kind of non-deneracy hypothesis on $g$. For example, if $g(x,y) = 0$ for all $x$ and $y$, then $g$ is symmetric. Now, let $V = E$. Then $V_1 = E$, but $Enotcong Eoplus E$ (for finite dimensional vector spaces).
    $endgroup$
    – Jason DeVito
    Oct 14 '15 at 14:18
















2












2








2


1



$begingroup$


Let $E$ be a finite-dimensional vector space over the field $k$ and let $g$ be a bilinear form which is symmetric, antisymmetric or hermitian. Let $V$ be a subspace of $E$. Let $V_1 = { x| x in E, g(x,a)=0 forall a in V}$. Prove that $E$ is a direct sum of $V$ and $V_1$. This result is used in the proof as well-known but unfortunately I can't find the proof of it. The proof I know is in the case of $g$ being inner product.










share|cite|improve this question











$endgroup$




Let $E$ be a finite-dimensional vector space over the field $k$ and let $g$ be a bilinear form which is symmetric, antisymmetric or hermitian. Let $V$ be a subspace of $E$. Let $V_1 = { x| x in E, g(x,a)=0 forall a in V}$. Prove that $E$ is a direct sum of $V$ and $V_1$. This result is used in the proof as well-known but unfortunately I can't find the proof of it. The proof I know is in the case of $g$ being inner product.







vector-spaces bilinear-form






share|cite|improve this question















share|cite|improve this question













share|cite|improve this question




share|cite|improve this question








edited Dec 11 '16 at 3:01









Gerry Myerson

147k8149302




147k8149302










asked Nov 7 '13 at 20:28









Nazar SerdyukNazar Serdyuk

1247




1247








  • 2




    $begingroup$
    Well, if you can't "find the proof" come up with your own! Or at least try...
    $endgroup$
    – DonAntonio
    Nov 7 '13 at 20:29






  • 1




    $begingroup$
    This is false as stated, you need some kind of non-deneracy hypothesis on $g$. For example, if $g(x,y) = 0$ for all $x$ and $y$, then $g$ is symmetric. Now, let $V = E$. Then $V_1 = E$, but $Enotcong Eoplus E$ (for finite dimensional vector spaces).
    $endgroup$
    – Jason DeVito
    Oct 14 '15 at 14:18
















  • 2




    $begingroup$
    Well, if you can't "find the proof" come up with your own! Or at least try...
    $endgroup$
    – DonAntonio
    Nov 7 '13 at 20:29






  • 1




    $begingroup$
    This is false as stated, you need some kind of non-deneracy hypothesis on $g$. For example, if $g(x,y) = 0$ for all $x$ and $y$, then $g$ is symmetric. Now, let $V = E$. Then $V_1 = E$, but $Enotcong Eoplus E$ (for finite dimensional vector spaces).
    $endgroup$
    – Jason DeVito
    Oct 14 '15 at 14:18










2




2




$begingroup$
Well, if you can't "find the proof" come up with your own! Or at least try...
$endgroup$
– DonAntonio
Nov 7 '13 at 20:29




$begingroup$
Well, if you can't "find the proof" come up with your own! Or at least try...
$endgroup$
– DonAntonio
Nov 7 '13 at 20:29




1




1




$begingroup$
This is false as stated, you need some kind of non-deneracy hypothesis on $g$. For example, if $g(x,y) = 0$ for all $x$ and $y$, then $g$ is symmetric. Now, let $V = E$. Then $V_1 = E$, but $Enotcong Eoplus E$ (for finite dimensional vector spaces).
$endgroup$
– Jason DeVito
Oct 14 '15 at 14:18






$begingroup$
This is false as stated, you need some kind of non-deneracy hypothesis on $g$. For example, if $g(x,y) = 0$ for all $x$ and $y$, then $g$ is symmetric. Now, let $V = E$. Then $V_1 = E$, but $Enotcong Eoplus E$ (for finite dimensional vector spaces).
$endgroup$
– Jason DeVito
Oct 14 '15 at 14:18












1 Answer
1






active

oldest

votes


















0












$begingroup$

Hint in the finite dimensional case. Let $;n=dim_kE=n;$ and take a basis of $;V_1;,;{v_1,..,v_k}$ . Complete this to a basis of the whole space: $;{v_1,..,v_k,v_{k+1},...,v_n};$ . Using Gram-Schmidt we can assume this is an orthonormal basis, and then



$${v_{k+1},...,v_n};;text{is a basis of};;V$$



In fact, I can't see how this is really different from the usual proof for inner products
...






share|cite|improve this answer











$endgroup$













  • $begingroup$
    Don't we need to follow a Gram-Schmidt-like process to have ${v_{k+1},ldots,v_n}subset V$?
    $endgroup$
    – Jonathan Y.
    Nov 7 '13 at 20:37










  • $begingroup$
    No...why would we?
    $endgroup$
    – DonAntonio
    Nov 7 '13 at 20:38










  • $begingroup$
    Well, if $Vsubsetmathbb{R}^2$ is $Span{(1,0)}$, and we take the standard inner product, then ${(0,1)}$ is a basis for $V_1$. One could complete it to a basis for $mathbb{R}^2$ in the following way: ${(0,1),(1,1)}$, but naturally $(1,1)notin V$.
    $endgroup$
    – Jonathan Y.
    Nov 7 '13 at 20:44












  • $begingroup$
    I think you have a point there, @JonathanY. I shall edit later, perhaps...or delete as it is late now. Thanks.
    $endgroup$
    – DonAntonio
    Nov 7 '13 at 20:48











Your Answer





StackExchange.ifUsing("editor", function () {
return StackExchange.using("mathjaxEditing", function () {
StackExchange.MarkdownEditor.creationCallbacks.add(function (editor, postfix) {
StackExchange.mathjaxEditing.prepareWmdForMathJax(editor, postfix, [["$", "$"], ["\\(","\\)"]]);
});
});
}, "mathjax-editing");

StackExchange.ready(function() {
var channelOptions = {
tags: "".split(" "),
id: "69"
};
initTagRenderer("".split(" "), "".split(" "), channelOptions);

StackExchange.using("externalEditor", function() {
// Have to fire editor after snippets, if snippets enabled
if (StackExchange.settings.snippets.snippetsEnabled) {
StackExchange.using("snippets", function() {
createEditor();
});
}
else {
createEditor();
}
});

function createEditor() {
StackExchange.prepareEditor({
heartbeatType: 'answer',
autoActivateHeartbeat: false,
convertImagesToLinks: true,
noModals: true,
showLowRepImageUploadWarning: true,
reputationToPostImages: 10,
bindNavPrevention: true,
postfix: "",
imageUploader: {
brandingHtml: "Powered by u003ca class="icon-imgur-white" href="https://imgur.com/"u003eu003c/au003e",
contentPolicyHtml: "User contributions licensed under u003ca href="https://creativecommons.org/licenses/by-sa/3.0/"u003ecc by-sa 3.0 with attribution requiredu003c/au003e u003ca href="https://stackoverflow.com/legal/content-policy"u003e(content policy)u003c/au003e",
allowUrls: true
},
noCode: true, onDemand: true,
discardSelector: ".discard-answer"
,immediatelyShowMarkdownHelp:true
});


}
});














draft saved

draft discarded


















StackExchange.ready(
function () {
StackExchange.openid.initPostLogin('.new-post-login', 'https%3a%2f%2fmath.stackexchange.com%2fquestions%2f556066%2fvector-space-is-decomposed-into-direct-sum-of-its-subspace-and-its-orthogonal-co%23new-answer', 'question_page');
}
);

Post as a guest















Required, but never shown

























1 Answer
1






active

oldest

votes








1 Answer
1






active

oldest

votes









active

oldest

votes






active

oldest

votes









0












$begingroup$

Hint in the finite dimensional case. Let $;n=dim_kE=n;$ and take a basis of $;V_1;,;{v_1,..,v_k}$ . Complete this to a basis of the whole space: $;{v_1,..,v_k,v_{k+1},...,v_n};$ . Using Gram-Schmidt we can assume this is an orthonormal basis, and then



$${v_{k+1},...,v_n};;text{is a basis of};;V$$



In fact, I can't see how this is really different from the usual proof for inner products
...






share|cite|improve this answer











$endgroup$













  • $begingroup$
    Don't we need to follow a Gram-Schmidt-like process to have ${v_{k+1},ldots,v_n}subset V$?
    $endgroup$
    – Jonathan Y.
    Nov 7 '13 at 20:37










  • $begingroup$
    No...why would we?
    $endgroup$
    – DonAntonio
    Nov 7 '13 at 20:38










  • $begingroup$
    Well, if $Vsubsetmathbb{R}^2$ is $Span{(1,0)}$, and we take the standard inner product, then ${(0,1)}$ is a basis for $V_1$. One could complete it to a basis for $mathbb{R}^2$ in the following way: ${(0,1),(1,1)}$, but naturally $(1,1)notin V$.
    $endgroup$
    – Jonathan Y.
    Nov 7 '13 at 20:44












  • $begingroup$
    I think you have a point there, @JonathanY. I shall edit later, perhaps...or delete as it is late now. Thanks.
    $endgroup$
    – DonAntonio
    Nov 7 '13 at 20:48
















0












$begingroup$

Hint in the finite dimensional case. Let $;n=dim_kE=n;$ and take a basis of $;V_1;,;{v_1,..,v_k}$ . Complete this to a basis of the whole space: $;{v_1,..,v_k,v_{k+1},...,v_n};$ . Using Gram-Schmidt we can assume this is an orthonormal basis, and then



$${v_{k+1},...,v_n};;text{is a basis of};;V$$



In fact, I can't see how this is really different from the usual proof for inner products
...






share|cite|improve this answer











$endgroup$













  • $begingroup$
    Don't we need to follow a Gram-Schmidt-like process to have ${v_{k+1},ldots,v_n}subset V$?
    $endgroup$
    – Jonathan Y.
    Nov 7 '13 at 20:37










  • $begingroup$
    No...why would we?
    $endgroup$
    – DonAntonio
    Nov 7 '13 at 20:38










  • $begingroup$
    Well, if $Vsubsetmathbb{R}^2$ is $Span{(1,0)}$, and we take the standard inner product, then ${(0,1)}$ is a basis for $V_1$. One could complete it to a basis for $mathbb{R}^2$ in the following way: ${(0,1),(1,1)}$, but naturally $(1,1)notin V$.
    $endgroup$
    – Jonathan Y.
    Nov 7 '13 at 20:44












  • $begingroup$
    I think you have a point there, @JonathanY. I shall edit later, perhaps...or delete as it is late now. Thanks.
    $endgroup$
    – DonAntonio
    Nov 7 '13 at 20:48














0












0








0





$begingroup$

Hint in the finite dimensional case. Let $;n=dim_kE=n;$ and take a basis of $;V_1;,;{v_1,..,v_k}$ . Complete this to a basis of the whole space: $;{v_1,..,v_k,v_{k+1},...,v_n};$ . Using Gram-Schmidt we can assume this is an orthonormal basis, and then



$${v_{k+1},...,v_n};;text{is a basis of};;V$$



In fact, I can't see how this is really different from the usual proof for inner products
...






share|cite|improve this answer











$endgroup$



Hint in the finite dimensional case. Let $;n=dim_kE=n;$ and take a basis of $;V_1;,;{v_1,..,v_k}$ . Complete this to a basis of the whole space: $;{v_1,..,v_k,v_{k+1},...,v_n};$ . Using Gram-Schmidt we can assume this is an orthonormal basis, and then



$${v_{k+1},...,v_n};;text{is a basis of};;V$$



In fact, I can't see how this is really different from the usual proof for inner products
...







share|cite|improve this answer














share|cite|improve this answer



share|cite|improve this answer








edited Dec 30 '17 at 22:21

























answered Nov 7 '13 at 20:34









DonAntonioDonAntonio

179k1494233




179k1494233












  • $begingroup$
    Don't we need to follow a Gram-Schmidt-like process to have ${v_{k+1},ldots,v_n}subset V$?
    $endgroup$
    – Jonathan Y.
    Nov 7 '13 at 20:37










  • $begingroup$
    No...why would we?
    $endgroup$
    – DonAntonio
    Nov 7 '13 at 20:38










  • $begingroup$
    Well, if $Vsubsetmathbb{R}^2$ is $Span{(1,0)}$, and we take the standard inner product, then ${(0,1)}$ is a basis for $V_1$. One could complete it to a basis for $mathbb{R}^2$ in the following way: ${(0,1),(1,1)}$, but naturally $(1,1)notin V$.
    $endgroup$
    – Jonathan Y.
    Nov 7 '13 at 20:44












  • $begingroup$
    I think you have a point there, @JonathanY. I shall edit later, perhaps...or delete as it is late now. Thanks.
    $endgroup$
    – DonAntonio
    Nov 7 '13 at 20:48


















  • $begingroup$
    Don't we need to follow a Gram-Schmidt-like process to have ${v_{k+1},ldots,v_n}subset V$?
    $endgroup$
    – Jonathan Y.
    Nov 7 '13 at 20:37










  • $begingroup$
    No...why would we?
    $endgroup$
    – DonAntonio
    Nov 7 '13 at 20:38










  • $begingroup$
    Well, if $Vsubsetmathbb{R}^2$ is $Span{(1,0)}$, and we take the standard inner product, then ${(0,1)}$ is a basis for $V_1$. One could complete it to a basis for $mathbb{R}^2$ in the following way: ${(0,1),(1,1)}$, but naturally $(1,1)notin V$.
    $endgroup$
    – Jonathan Y.
    Nov 7 '13 at 20:44












  • $begingroup$
    I think you have a point there, @JonathanY. I shall edit later, perhaps...or delete as it is late now. Thanks.
    $endgroup$
    – DonAntonio
    Nov 7 '13 at 20:48
















$begingroup$
Don't we need to follow a Gram-Schmidt-like process to have ${v_{k+1},ldots,v_n}subset V$?
$endgroup$
– Jonathan Y.
Nov 7 '13 at 20:37




$begingroup$
Don't we need to follow a Gram-Schmidt-like process to have ${v_{k+1},ldots,v_n}subset V$?
$endgroup$
– Jonathan Y.
Nov 7 '13 at 20:37












$begingroup$
No...why would we?
$endgroup$
– DonAntonio
Nov 7 '13 at 20:38




$begingroup$
No...why would we?
$endgroup$
– DonAntonio
Nov 7 '13 at 20:38












$begingroup$
Well, if $Vsubsetmathbb{R}^2$ is $Span{(1,0)}$, and we take the standard inner product, then ${(0,1)}$ is a basis for $V_1$. One could complete it to a basis for $mathbb{R}^2$ in the following way: ${(0,1),(1,1)}$, but naturally $(1,1)notin V$.
$endgroup$
– Jonathan Y.
Nov 7 '13 at 20:44






$begingroup$
Well, if $Vsubsetmathbb{R}^2$ is $Span{(1,0)}$, and we take the standard inner product, then ${(0,1)}$ is a basis for $V_1$. One could complete it to a basis for $mathbb{R}^2$ in the following way: ${(0,1),(1,1)}$, but naturally $(1,1)notin V$.
$endgroup$
– Jonathan Y.
Nov 7 '13 at 20:44














$begingroup$
I think you have a point there, @JonathanY. I shall edit later, perhaps...or delete as it is late now. Thanks.
$endgroup$
– DonAntonio
Nov 7 '13 at 20:48




$begingroup$
I think you have a point there, @JonathanY. I shall edit later, perhaps...or delete as it is late now. Thanks.
$endgroup$
– DonAntonio
Nov 7 '13 at 20:48


















draft saved

draft discarded




















































Thanks for contributing an answer to Mathematics Stack Exchange!


  • Please be sure to answer the question. Provide details and share your research!

But avoid



  • Asking for help, clarification, or responding to other answers.

  • Making statements based on opinion; back them up with references or personal experience.


Use MathJax to format equations. MathJax reference.


To learn more, see our tips on writing great answers.




draft saved


draft discarded














StackExchange.ready(
function () {
StackExchange.openid.initPostLogin('.new-post-login', 'https%3a%2f%2fmath.stackexchange.com%2fquestions%2f556066%2fvector-space-is-decomposed-into-direct-sum-of-its-subspace-and-its-orthogonal-co%23new-answer', 'question_page');
}
);

Post as a guest















Required, but never shown





















































Required, but never shown














Required, but never shown












Required, but never shown







Required, but never shown

































Required, but never shown














Required, but never shown












Required, but never shown







Required, but never shown







Popular posts from this blog

How do I know what Microsoft account the skydrive app is syncing to?

When does type information flow backwards in C++?

Grease: Live!